How Many Ways Can Martians, Vesuvians, and Jovians Line Up?

  • Thread starter Thread starter RedSunflowers32
  • Start date Start date
Click For Summary
The discussion revolves around calculating the number of ways to arrange 5 distinct Martians, 10 distinct Vesuvians, and 8 distinct Jovians in a line, ensuring that no two Martians stand together. Participants are confused about how to approach the problem, particularly in incorporating the constraints of the Martians' placement. One suggested method involves arranging the Vesuvians and Jovians first, then filling in the Martians in the gaps created. The calculations presented include factorials for permutations, but there is uncertainty about the correct equation to arrive at the final answer. The overall consensus indicates a need for clarity on the combination and permutation principles involved in the problem.
RedSunflowers32
Messages
6
Reaction score
0
In how many ways can 5 distinct Martians, ten distinct Vesuvians and eight distinct Jovians wait together in line if no two Martains stand together>



2. 10_p_5 = 10! & 5! but what about 8? and 2?



3. 10! = 10*9*8*7*6*5*4*3*2*1
8! = 8*7*6*5*4*3*2*1
5! = 5*4*3*2*1
10*9 = 90/2 = 45?



I am so lost.. I am not a programmer..but required to take this class for my major! Please anyone help me out??
 
Physics news on Phys.org
consider a line of vesuvians and jovians and fill the martians into the gaps. That'll be the total possible ways of fitting in Martians so that no two matians stay together.
 
hummm

I did that one already and I didn't like the answer. But thanks.
 
RedSunflowers32 said:
I did that one already and I didn't like the answer.
what does that mean :confused:
 
It means I did the chain with every 3rd position there is a Martian, but that does not tell me how to write the equation to present the proper answer. If I factor out each element 5! = 120 * 10! = 3628800 *8! =40320
which gives me an answer of 1.5801827328E15.
Since we are working with combinations and Premutations I'm lost on which way to do the problem.
 
Question: A clock's minute hand has length 4 and its hour hand has length 3. What is the distance between the tips at the moment when it is increasing most rapidly?(Putnam Exam Question) Answer: Making assumption that both the hands moves at constant angular velocities, the answer is ## \sqrt{7} .## But don't you think this assumption is somewhat doubtful and wrong?

Similar threads

  • · Replies 2 ·
Replies
2
Views
1K
  • · Replies 3 ·
Replies
3
Views
2K
  • · Replies 2 ·
Replies
2
Views
1K
Replies
2
Views
1K
  • · Replies 5 ·
Replies
5
Views
4K
  • · Replies 9 ·
Replies
9
Views
2K
Replies
15
Views
4K
Replies
14
Views
2K
  • · Replies 11 ·
Replies
11
Views
2K
Replies
2
Views
2K